Diễn Đàn MathScopeDiễn Đàn MathScope
  Diễn Đàn MathScope
Ghi Danh Hỏi/Ðáp Community Lịch

Go Back   Diễn Đàn MathScope > Sơ Cấp > Đại Số và Lượng Giác

News & Announcements

Ngoài một số quy định đã được nêu trong phần Quy định của Ghi Danh , mọi người tranh thủ bỏ ra 5 phút để đọc thêm một số Quy định sau để khỏi bị treo nick ở MathScope nhé !

* Nội quy MathScope.Org

* Một số quy định chung !

* Quy định về việc viết bài trong diễn đàn MathScope

* Nếu bạn muốn gia nhập đội ngũ BQT thì vui lòng tham gia tại đây

* Những câu hỏi thường gặp

* Về việc viết bài trong Box Đại học và Sau đại học


Trả lời Gởi Ðề Tài Mới
 
Ðiều Chỉnh Xếp Bài
Old 12-06-2011, 10:15 AM   #1
company
+Thành Viên+
 
company's Avatar
 
Tham gia ngày: Jun 2011
Bài gởi: 39
Thanks: 92
Thanked 28 Times in 16 Posts
Bài 105: Cho các số thực không âm a, b, c thỏa mãn không có đồng thời hai số nào bằng 0. Chứng minh rằng:
$\sum \left ( \frac{a}{b+c} \right )^{2}+\frac{1}{2}\geq \frac{5}{4}.\frac{a^{2}+b^{2}+c^{2}}{ab+bc+ca} $

Bài này giải như sau:

Áp dụng Cauchy Schwarz ta chỉ cần CM
$\frac{(a^2+b^2+c^2)^2}{a^2(b+c)^2+b^2(c+a)^2+c^2(a +b)^2} +\frac{1}{2} \geq \frac{5}{4}.\frac{a^2+b^2+c^2}{ab+bc+ca} (1) $

Chuẩn hóa $a+b+c=1 $ ,
Đặt $q=ab+bc+ca, abc=r $
Khi đó (1) trở thành :
$\frac{(1-2q)^2}{q^2-r} +6 \geq \frac{5}{2q} $
Xét với $0 <q<\frac{1}{4} $
$\frac{(1-2q)^2}{q^2-r} +6 \geq \frac{(1-2q)^2}{q^2} +6 =\frac{5}{2q}+\frac{(2-5q)(1-4q)}{2q^2} \geq \frac{5}{2q} $
Xét với $\frac{1}{4} \leq q \leq \frac{1}{3} $
$\frac{(1-2q)^2}{q^2-r} +6 \geq \frac{(1-2q)^2}{q^2 -\frac{(4q-1)(1-q)}{6}}+6 =\frac{5}{2q}+...... \geq \frac{5}{2q} $
DONE!!!!

Bài 106
Cho các số thực không âm thỏa mãn $(a+b)(b+c)(c+a) >0 $
CMR
$\frac{1}{(b+c)^2}+\frac{1}{(c+a)^2}+\frac{1}{(a+b) ^2} \geq \frac{9}{4(ab+bc+ca)} $
[RIGHT][I][B]Nguồn: MathScope.ORG[/B][/I][/RIGHT]
 
__________________
Never say never!

thay đổi nội dung bởi: company, 12-06-2011 lúc 10:19 AM
company is offline   Trả Lời Với Trích Dẫn
Old 11-06-2011, 06:10 PM   #2
magician_14312
Moderator
 
magician_14312's Avatar
 
Tham gia ngày: Jan 2011
Đến từ: Solar System
Bài gởi: 367
Thanks: 201
Thanked 451 Times in 220 Posts
Trích:
Nguyên văn bởi hien123 View Post
Bài 104:Cho các số thực không âm a, b, c thỏa mãn $a+b+c> 0 $. Chứng minh rằng:
$\sum \frac{a^{2}}{2a^{2}+(b+c)^{2}}\leq \frac{2}{3}
$
Bất đẳng thức đã cho tương đương với: $\sum \frac{a^{2}}{4a^{2}+2(b+c)^{2}}\leq \frac{1}{3} $
Sử dụng bđt Cauchy-Schwarz ta có:
$\frac{9}{4a^{2}+2(b+c)^{2}}=\frac{(2+1)^2}{2(a^2+b ^2+c^2)+2a^2+4bc}\leq \frac{4}{2(a^2+b^2+c^2)}+\frac{1}{2a^2+4bc}\leq \frac{2}{a^2+b^2+c^2}+\frac{1}{2a^2+bc} $
Do đó:
$\sum \frac{a^2}{4a^{2}+2(b+c)^{2}}\leq \frac{1}{9}\left ( \sum \frac{2a^2}{a^2+b^2+c^2}+\sum \frac{a^2}{2a^2+bc} \right ) $
hay $\sum \frac{a^2}{4a^{2}+2(b+c)^{2}}\leq \frac{1}{9}\left ( 2+\sum \frac{a^2}{2a^2+bc} \right ) $
Sử dụng 1 kết quả quen thuộc là $\frac{a^2}{2a^2+bc}+\frac{b^2}{2b^2+ca}+\frac{c^2} {2c^2+ab}\leq 1 $, ta có:
$\sum \frac{a^2}{4a^{2}+2(b+c)^{2}}\leq \frac{3}{9}=\frac{1}{3} $ (đpcm).
Dấu đẳng thức xảy ra khi $a=b;c=0 $ và các hoán vị.
[RIGHT][I][B]Nguồn: MathScope.ORG[/B][/I][/RIGHT]
 
magician_14312 is offline   Trả Lời Với Trích Dẫn
The Following User Says Thank You to magician_14312 For This Useful Post:
company (11-06-2011)
Old 22-06-2011, 05:34 PM   #3
Nguyenhuyen_AG
+Thành Viên+
 
Nguyenhuyen_AG's Avatar
 
Tham gia ngày: Apr 2010
Bài gởi: 300
Thanks: 35
Thanked 307 Times in 151 Posts
Bài 107. Với $a,b,c $ là các số thực dương sao cho $\left (a-\sqrt[3]{abc} \right )\left ( b-\sqrt[3]{abc} \right )\left ( c-\sqrt[3]{abc} \right )\le0 $. Chứng minh bất đẳng thức

$\frac{a+b+c}{\sqrt[3]{abc}}\ge\frac{a+b}{b+c}+\frac{b+c}{a+b}+1 $

Bài 108. Nếu $a,b,c $ là các số thực dương, thì

$\frac{a}{b}+\frac{b}{c}+\frac{c}{a}\ge \frac{a+b+c}{\sqrt[3]{abc}} $

[RIGHT][I][B]Nguồn: MathScope.ORG[/B][/I][/RIGHT]
 
__________________
Nguyen Van Huyen
Ho Chi Minh City University of Transport

thay đổi nội dung bởi: Nguyenhuyen_AG, 22-06-2011 lúc 05:52 PM
Nguyenhuyen_AG is offline   Trả Lời Với Trích Dẫn
Old 22-06-2011, 07:45 PM   #4
ilovehien95
+Thành Viên+
 
Tham gia ngày: Jun 2011
Bài gởi: 76
Thanks: 142
Thanked 13 Times in 8 Posts
Trích:
Nguyên văn bởi Nguyenhuyen_AG View Post
Bài 107. Với $a,b,c $ là các số thực dương sao cho $\left (a-\sqrt[3]{abc} \right )\left ( b-\sqrt[3]{abc} \right )\left ( c-\sqrt[3]{abc} \right )\le0 $. Chứng minh bất đẳng thức

$\frac{a+b+c}{\sqrt[3]{abc}}\ge\frac{a+b}{b+c}+\frac{b+c}{a+b}+1 $

Bài 108. Nếu $a,b,c $ là các số thực dương, thì

$\frac{a}{b}+\frac{b}{c}+\frac{c}{a}\ge \frac{a+b+c}{\sqrt[3]{abc}} $
Bài 108

[RIGHT][I][B]Nguồn: MathScope.ORG[/B][/I][/RIGHT]
 
__________________
Listen to the rhymth of the falling rain. Tellling me what a fool i've been........I CANT love another when my heart somewhere faraway
ilovehien95 is offline   Trả Lời Với Trích Dẫn
Old 23-06-2011, 01:21 PM   #5
Nguyenhuyen_AG
+Thành Viên+
 
Nguyenhuyen_AG's Avatar
 
Tham gia ngày: Apr 2010
Bài gởi: 300
Thanks: 35
Thanked 307 Times in 151 Posts
Trích:
Nguyên văn bởi ilovehien95 View Post
Bài 108
Anh cần lời giải bằng Cauchy-Schwarz mà.
[RIGHT][I][B]Nguồn: MathScope.ORG[/B][/I][/RIGHT]
 
__________________
Nguyen Van Huyen
Ho Chi Minh City University of Transport
Nguyenhuyen_AG is offline   Trả Lời Với Trích Dẫn
Old 09-08-2011, 03:12 PM   #6
vthiep94
+Thành Viên+
 
Tham gia ngày: Dec 2009
Bài gởi: 197
Thanks: 185
Thanked 49 Times in 31 Posts
Bài 130. Bất đẳng thức cũ (Iran 1994 thì phải)
Cm với a,b,c không âm sao cho ab+bc+ac > 0
$ (ab+bc+ac)(\dfrac{1}{(b+c)^2}+\dfrac{1}{(a+c)^2}+ \dfrac{1}{(a+b)^2}) \ge \dfrac{9}{4} $

p/s : bài này không biết có thể dùng phương pháp sắp xếp biến không nhỉ ?
Mình làm mãi không được. Mong các bạn chỉ giáo
[RIGHT][I][B]Nguồn: MathScope.ORG[/B][/I][/RIGHT]
 

thay đổi nội dung bởi: batigoal, 09-08-2011 lúc 04:00 PM Lý do: Phải đánh số theo thứ tự
vthiep94 is offline   Trả Lời Với Trích Dẫn
Old 09-08-2011, 03:58 PM   #7
je.triste
+Thành Viên+
 
je.triste's Avatar
 
Tham gia ngày: Feb 2011
Bài gởi: 358
Thanks: 437
Thanked 186 Times in 128 Posts
Trích:
Nguyên văn bởi vthiep94 View Post
Bất đẳng thức cũ (Iran 1994 thì phải)
Cm với a,b,c không âm sao cho ab+bc+ac > 0
$ (ab+bc+ac)(\dfrac{1}{(b+c)^2}+\dfrac{1}{(a+c)^2}+ \dfrac{1}{(a+b)^2}) \ge \dfrac{9}{4} $

p/s : bài này không biết có thể dùng phương pháp sắp xếp biến không nhỉ ?
Mình làm mãi không được. Mong các bạn chỉ giáo

@Ruango: Em chỉnh lại Latex.
Đây là topic BĐT Cauchy-SChawrz nên dùng C_S để chứng minh. nêú post phương pháp khác xin vui lòng cho vào gợi ý

[RIGHT][I][B]Nguồn: MathScope.ORG[/B][/I][/RIGHT]
 
__________________
Mathscope...
Giá trị đích thực của sự cho đi không nằm ở món quà lớn hay nhỏ, mà nằm ở tầm lòng của người cho!

thay đổi nội dung bởi: je.triste, 09-08-2011 lúc 04:10 PM
je.triste is offline   Trả Lời Với Trích Dẫn
The Following 2 Users Say Thank You to je.triste For This Useful Post:
nhox12764 (23-11-2011), vthiep94 (09-08-2011)
Old 23-06-2011, 01:59 PM   #8
trung65
+Thành Viên+
 
Tham gia ngày: Sep 2010
Đến từ: Tp HCM
Bài gởi: 46
Thanks: 31
Thanked 48 Times in 24 Posts
Trích:
Nguyên văn bởi Nguyenhuyen_AG View Post
Bài 107. Với $a,b,c $ là các số thực dương sao cho $\left (a-\sqrt[3]{abc} \right )\left ( b-\sqrt[3]{abc} \right )\left ( c-\sqrt[3]{abc} \right )\le0 $. Chứng minh bất đẳng thức

$\frac{a+b+c}{\sqrt[3]{abc}}\ge\frac{a+b}{b+c}+\frac{b+c}{a+b}+1 $
Ta viết lại bđt dưới dạng:
$\sum\frac{a^2}{a\sqrt[3]{abc}} + \frac{b^2}{b^2}\geq \frac{(a+b+b+c)^2}{(b+c)(b+a)} $
Áp dụng bđt CS ta có :
$\sum\frac{a^2}{a\sqrt[3]{abc}} + \frac{b^2}{b^2}\geq \frac{(a+b+b+c)^2}{(a+b+c)\sqrt[3]{abc}+b^2} $
Vậy ta cần chứng minh:
$(b+c)(b+a)\geq(a+b+c)\sqrt[3]{abc}+b^2 $(tương đương với giả thiết)
[RIGHT][I][B]Nguồn: MathScope.ORG[/B][/I][/RIGHT]
 
trung65 is offline   Trả Lời Với Trích Dẫn
The Following 3 Users Say Thank You to trung65 For This Useful Post:
ilovehien95 (26-06-2011), leviethai (23-06-2011), MathForLife (26-06-2011)
Old 27-06-2011, 03:06 PM   #9
MathForLife
+Thành Viên+
 
Tham gia ngày: Sep 2010
Đến từ: CT force
Bài gởi: 731
Thanks: 603
Thanked 425 Times in 212 Posts
Trích:
Nguyên văn bởi Nguyenhuyen_AG View Post
Bài 108. Nếu $a,b,c $ là các số thực dương, thì

$\frac{a}{b}+\frac{b}{c}+\frac{c}{a}\ge \frac{a+b+c}{\sqrt[3]{abc}} $
Đặt $\frac{a}{b}=x^3; \frac{b}{c}=y^3; \frac{c}{a}=z^3 $, khi đó dễ thấy rằng vế phải của bất đẳng thức bằng $x^2y+y^z+z^2x $. Ta chuyển bất đẳng thức về dạng:

$x^3+y^3+z^3\ge x^2y+y^2z+z^2x $
Đến đây sử dụng bất đẳng thức Cauchy-Schwarz như sau:

$\left [ (xy^2+yz^2+zx^2)(x^3+y^3+z^3) \right ]^2\ge (x^2y+y^2z+z^2x)^2 $

$(x^2y+y^2z+z^2x)(y^3+z^3+x^3)\ge (xy^2+yz^2+zx^2)^2 $
Nhân 2 bất đẳng thức trên vế theo vế, sau đó rút gọn cho $xy^2+yz^2+zx^2 $ và lấy căn bậc ba ta đươc đpcm.
Đăng thức chỉ xảy ra khi $x=y=z=1 $ hay $a=b=c $.
[RIGHT][I][B]Nguồn: MathScope.ORG[/B][/I][/RIGHT]
 
MathForLife is offline   Trả Lời Với Trích Dẫn
Old 27-06-2011, 05:21 PM   #10
Aponium
+Thành Viên+
 
Aponium's Avatar
 
Tham gia ngày: Jul 2010
Bài gởi: 40
Thanks: 79
Thanked 11 Times in 11 Posts
Bài 109: Cho các số thực dương a,b,c. Tìm giá trị nhỏ nhất của biểu thức:
$P= \frac{a+b}{a+b+c} + \frac{b+c}{b+c+4a} + \frac{c+a}{c+a+16b} $
Giải theo CS các bạn nhé!!!
[RIGHT][I][B]Nguồn: MathScope.ORG[/B][/I][/RIGHT]
 
Aponium is offline   Trả Lời Với Trích Dẫn
Old 30-06-2011, 10:48 AM   #11
batigoal
Super Moderator
 
batigoal's Avatar
 
Tham gia ngày: Jul 2010
Đến từ: Hà Nội
Bài gởi: 2,895
Thanks: 382
Thanked 2,968 Times in 1,295 Posts
Bài 110.Cho ba số dương$a,b,c $.Chứng minh rằng:

$\left.\sqrt{\frac{a^{5}}{2a^{2}b+3c^{3}}}+ \sqrt{\frac{b^{5}}{2b^{2}c+3a^{3}}}+\sqrt{\frac{c^ {5}}{2c^{2}a+3b^{3}}}\geq\sqrt{\frac{3(a^{2}+b^{2} +c^{2})}{5}}\right. $

[RIGHT][I][B]Nguồn: MathScope.ORG[/B][/I][/RIGHT]
 
__________________
“ Sức mạnh của tri thức là sự chia sẻ tri thức”

[Only registered and activated users can see links. ]
batigoal is offline   Trả Lời Với Trích Dẫn
Old 30-06-2011, 11:21 AM   #12
daiduong1095
+Thành Viên+
 
daiduong1095's Avatar
 
Tham gia ngày: Sep 2010
Đến từ: CVP-Math
Bài gởi: 287
Thanks: 13
Thanked 210 Times in 112 Posts
Gửi tin nhắn qua Yahoo chát tới daiduong1095
Trích:
Nguyên văn bởi batigoal View Post
Bài 110.Cho ba số dương$a,b,c $.Chứng minh rằng:

$\left.\sqrt{\frac{a^{5}}{2a^{2}b+3c^{3}}}+ \sqrt{\frac{b^{5}}{2b^{2}c+3a^{3}}}+\sqrt{\frac{c^ {5}}{2c^{2}a+3b^{3}}}\geq\sqrt{\frac{3(a^{2}+b^{2} +c^{2})}{5}}\right. $
Dùng bổ đề sau:$(a^2+b^2+c^2)^2\geq 3\left(a^{3}b+b^{3}c+c^{3}a \right) $.Áp dụng BDT Cauchy-Schwarz ta có:
VT$\geq \frac{\left(a^{2}+b^{2}+c^{2} \right)^{2}}{a\sqrt{2a^{3}b+3c^{3}a}+b\sqrt{2b^{3} c+3a^{3}b}+c\sqrt{2c^{3}a+3a^{3}b}}\geq \frac{\left(a^{2}+b^{2}+c^{2} \right)^{2}}{\sqrt{5\left(a^{2}+b^{2}+c^{2} \right)\left(a^{3}b+b^{3}c+c^{3}a \right)}} $.
Từ đó theo bổ đề suy ra đpcm.
[RIGHT][I][B]Nguồn: MathScope.ORG[/B][/I][/RIGHT]
 
__________________

thay đổi nội dung bởi: daiduong1095, 30-06-2011 lúc 11:35 AM
daiduong1095 is offline   Trả Lời Với Trích Dẫn
The Following 2 Users Say Thank You to daiduong1095 For This Useful Post:
ilovehien95 (05-07-2011), levanquy (11-03-2012)
Old 05-07-2011, 01:21 PM   #13
Nguyenhuyen_AG
+Thành Viên+
 
Nguyenhuyen_AG's Avatar
 
Tham gia ngày: Apr 2010
Bài gởi: 300
Thanks: 35
Thanked 307 Times in 151 Posts
Bài 111. Với $a,b,c $ là các số dương. Chứng minh các bất đẳng thức sau

$\frac{a}{b+c}.\left ( \frac{b}{c} \right )^2+\frac{b}{c+a}.\left ( \frac{c}{a} \right )^2+\frac{c}{a+b}.\left ( \frac{a}{b} \right )^2\ge \frac{3}{2}.\frac{a^2+b^2+c^2}{ab+bc+ca} $

$\frac{ab}{c\sqrt{a+b}}+\frac{bc}{a\sqrt{b+c}}+ \frac{ca}{b\sqrt{c+a}}\ge \sqrt{\frac{3}{2}.(a+b+c)} $

[RIGHT][I][B]Nguồn: MathScope.ORG[/B][/I][/RIGHT]
 
__________________
Nguyen Van Huyen
Ho Chi Minh City University of Transport
Nguyenhuyen_AG is offline   Trả Lời Với Trích Dẫn
Old 05-07-2011, 01:32 PM   #14
sang89
+Thành Viên Danh Dự+
 
Tham gia ngày: Mar 2010
Đến từ: Heaven
Bài gởi: 887
Thanks: 261
Thanked 463 Times in 331 Posts
Trích:
Nguyên văn bởi Nguyenhuyen_AG View Post
Bài 111. Với $a,b,c $ là các số dương. Chứng minh các bất đẳng thức sau



$\frac{ab}{c\sqrt{a+b}}+\frac{bc}{a\sqrt{b+c}}+ \frac{ca}{b\sqrt{c+a}}\ge \sqrt{\frac{3}{2}.(a+b+c)} $
Áp dụng Holder:

$\left(\sum \frac{ab}{c\sqrt{ab}}\right)^2 \left(\sum abc^2(a+b)\right) \ge (ab+bc+ca)^3 $

Đo đóm ta cần chứng minh rằng:

$\frac{(ab+bc+ca)^3}{\sum abc^2(a+b)} \ge \frac{3}{2}(a+b+c) $

Chỉ cần chuẩn hóa $abc=1 $ thì đây là bất đẳng thức quen thuộc:

$(ab+bc+ca)^2 \ge 3abc(a+b+c) $
[RIGHT][I][B]Nguồn: MathScope.ORG[/B][/I][/RIGHT]
 

thay đổi nội dung bởi: sang89, 05-07-2011 lúc 01:34 PM
sang89 is offline   Trả Lời Với Trích Dẫn
Old 05-07-2011, 01:38 PM   #15
daiduong1095
+Thành Viên+
 
daiduong1095's Avatar
 
Tham gia ngày: Sep 2010
Đến từ: CVP-Math
Bài gởi: 287
Thanks: 13
Thanked 210 Times in 112 Posts
Gửi tin nhắn qua Yahoo chát tới daiduong1095
Trích:
Nguyên văn bởi Nguyenhuyen_AG View Post
Bài 111. Với $a,b,c $ là các số dương. Chứng minh các bất đẳng thức sau

$\frac{ab}{c\sqrt{a+b}}+\frac{bc}{a\sqrt{b+c}}+ \frac{ca}{b\sqrt{c+a}}\ge \sqrt{\frac{3}{2}.(a+b+c)} $
Không cần Holder mà C-S thôi:
VT $\ge \frac{(ab+bc+ca)^2}{abc\left(\sqrt{a+b}+\sqrt{b+c} +\sqrt{c+a} \right)} \ge \frac{3abc(a+b+c)}{abc\sqrt{3(a+b+b+c+c+a)}}= $ VP.dpcm.
------------------------------
Trích:
Nguyên văn bởi Nguyenhuyen_AG View Post
Bài 111. Với $a,b,c $ là các số dương. Chứng minh các bất đẳng thức sau
$\frac{a}{b+c}.\left ( \frac{b}{c} \right )^2+\frac{b}{c+a}.\left ( \frac{c}{a} \right )^2+\frac{c}{a+b}.\left ( \frac{a}{b} \right )^2\ge \frac{3}{2}.\frac{a^2+b^2+c^2}{ab+bc+ca} $
Dùng C-S ta có:
$\left(a(b+c)+b(c+a)+c(a+b) \right)\left(\frac{a}{b+c}.\left ( \frac{b}{c} \right )^2+\frac{b}{c+a}.\left ( \frac{c}{a} \right )^2+\frac{c}{a+b}.\left ( \frac{a}{b} \right )^2 \right)\ge \left(\frac{ab}{c}+\frac{bc}{a}+\frac{ca}{b} \right)^2 $
$\ge 3(\frac{ab}{c}.\frac{ca}{b}+\frac{ca}{b}.\frac{bc} {a}+\frac{bc}{a}.\frac{ab}{c})=3(a^2+b^2+c^2) $
DPCM!
[RIGHT][I][B]Nguồn: MathScope.ORG[/B][/I][/RIGHT]
 
__________________

thay đổi nội dung bởi: daiduong1095, 05-07-2011 lúc 02:04 PM Lý do: Tự động gộp bài
daiduong1095 is offline   Trả Lời Với Trích Dẫn
The Following 2 Users Say Thank You to daiduong1095 For This Useful Post:
ilovehien95 (05-07-2011), innocent (09-07-2011)
Trả lời Gởi Ðề Tài Mới

Bookmarks


Quuyền Hạn Của Bạn
You may not post new threads
You may not post replies
You may not post attachments
You may not edit your posts

BB code is Mở
Smilies đang Mở
[IMG] đang Mở
HTML đang Tắt

Chuyển đến


Múi giờ GMT. Hiện tại là 08:07 PM.


Powered by: vBulletin Copyright ©2000-2024, Jelsoft Enterprises Ltd.
Inactive Reminders By mathscope.org
[page compression: 146.14 k/164.67 k (11.25%)]